Lesson: Weaken the Argument Questions

Comment on Weaken the Argument Questions

gmat-admin's picture

Question link: https://gmatclub.com/forum/archaeologist-researchers-excavating-a-burial...

The argument's conclusion is that, since the human and cat were buried together, cats must have been domesticated around this time.

(D) In Cyprus, there are many burial sites dating from around 9,500 years ago in which the remains of wild animals appear to have been buried alongside human remains.
This weakens the argument since it suggests the cat may have been wild (i.e., not domesticated) when it was buried with the human.

(A) Archaeologists have not found any remains of stores of grain in the immediate vicinity of the burial site.
This answer doesn't weaken the argument nearly as much as answer choice D does.
First of all, it could be the case that there ARE remains of stores of grain in the immediate vicinity, but no one has found them yet.
Or there could be remains of stores of grain nearby (just not in the immediate vicinity).

More importantly, keep in mind that the the conclusion is based solely on the fact that the human and cat were buried together. The part about protecting the grain from mice is simply a benefit of having domesticated cats.

Thanks, it's clear now.

Hi Brent,
Can you please explain why D is not the right answer for this question?
https://gmatclub.com/forum/images-from-ground-based-telescopes-are-invariably-distorted-by-the-ea-208210.html

Hi Brent.
Good Evening.
In Question - https://gmatclub.com/forum/since-the-deregulation-of-airlines-delays-at-the-nation-s-increasingl-80613.html,
I thought conclusion is 'more of the takeoff and landing slots at the busiest airports must be allocated to commercial airlines' and started looking for the answer which weakens it, but when I saw that answer is 'A', it confused me as it doesn't weaken the conclusion as per my understanding. Can you please help?
gmat-admin's picture

Question link: https://gmatclub.com/forum/since-the-deregulation-of-airlines-delays-at-...

I would go a little further with the conclusion.
Why does the author want more takeoff/landing slots allocated to commercial airlines?
The author feels that these extra slots will help reduce delays at the airports.
So the conclusion is more like: Additional takeoff/landing slots for commercial airlines will help reduce delays.

(A) The major causes of delays at the nation's busiest airports are bad weather and overtaxed air traffic control equipment.
That suggests that most delays have nothing to do with takeoff/landing slots for commercial airlines.
For example, if an airport is closed all day due to fog, additional takeoff/landing slots for commercial airlines will have no impact on reducing delays.

Does that help?

Hi Brent. Good Evening.
In Question - https://gmatclub.com/forum/increased-use-of-incineration-is-sometimes-advocated-as-a-safe-way-to-201149.html,
I believe that conclusion is 'Since designs for proposed new incinerators include no additional means of preventing such releases, leaks will only become more prevalent if use of incineration increases', but I got confused between option 'A' & 'D' and marked 'D' as correct. Can you please explain this?
gmat-admin's picture

Question link: https://gmatclub.com/forum/increased-use-of-incineration-is-sometimes-ad...

Argument summary:
40 incidents last year at 2 existing incinerators
Designs for new incinerators have no additional ways to prevent leaks
Conclusion: More incineration will cause more leaks

The main idea here is that poor incineration technology CAUSES leaks.

(A) At the two incinerators at which leaks were reported, staff had had only cursory training on the proper procedures for incinerating chemical waste.
This answer choice weakens the idea that poor incineration technology CAUSES leaks. Instead, the leaks were caused by poorly-trained staff.
So, as long as staff members are well-trained, there's no reason to conclude that more incineration will cause more leaks

(D) The frequency of reports of unexpected releases of chemical agents at newly built incinerators is about the same as the frequency at older incinerators.
If the frequency of leaks is the same at both new and old incinerators, then we should expect that more incineration will cause more leaks.
This answer choice STRENGTHENS the argument (we want to weaken the argument)

Hi Brent. Good Evening.
In Question - https://gmatclub.com/forum/fast-food-restaurants-make-up-45-percent-of-all-restaurants-in-cantari-220525.html,
I am confused between option 'C' & 'D'. Can you please explain how to decode the correct answer in this type of question?
gmat-admin's picture

Question link: https://gmatclub.com/forum/fast-food-restaurants-make-up-45-percent-of-a...

Argument summary:
45% of restaurants are fast food
Older people less likely to eat fast food
Average age is gradually rising
Conclusion: The number of fast-food restaurants is likely to decrease.

(C) Many people who rarely eat in fast-food restaurants nevertheless eat regularly in restaurants.
This doesn't weaken the argument, since it has no bearing on the fact that older people less likely to eat fast food, and that there are more and more old people each year.

(D) The overall population of Canatria is growing steadily.
The passage tells us that the average AGE is GRADUALLY rising. This SUGGESTS that the NUMBER of young people is decreases and this will cause the number of fast food restaurants to decrease.
However, if the overall POPULATION is it growing STEADILY, then the NUMBER of young people might not decrease. If the NUMBER of young people doesn't decrease, then we shouldn't see a decrease in the number of fast-food restaurants.
This weakens the conclusion

Hi Brent. Good Evening.
In Question - https://gmatclub.com/forum/in-stenland-many-workers-have-been-complaining-that-they-cannot-220495.html,
I believe conclusion is 'If minimum wage is raised, unemployment will increase because employer will no longer be able to afford to employ as many workers', and I chose 'E' as answer. Can you please correct me and explain the right answer?
gmat-admin's picture

Question link: https://gmatclub.com/forum/in-stenland-many-workers-have-been-complainin...
Tricky question!!

Employers' conclusion: If we have to increase wages, we won't be able to employ as many workers.

(A) For any position with wages below a living wage, the difficulty of finding and retaining employees adds as much to employment costs as would raising wages.
This says, if you pay people poorly (below living wage), you end up spending a lot of money finding and retaining employees.
The answer choice goes even further to say that the cost of finding/retaining employees is EQUAL to the additional cost of raising wages.
So, if employers increase wages to living wages, the amount they save will offset the extra money required to raise wages.
This weakens the conclusion that increased wages means the company can't employ as many workers.

(E) Many employers who pay some workers only the minimum wage also pay other workers wages that are much higher than the minimum.
This information has no direct effect on the employers' conclusion that they won't be able to employ as many workers if they increase wages.

Hi Brent. Good Evening.
Questions like https://gmatclub.com/forum/a-new-law-gives-ownership-of-patents-documents-providing-exclusive-rig-95195.html,
are being very confusing and causing frustration because every other time the answer I chose is wrong. Please help me out. In this case, I chose option 'E' but found 'D' as correct answer..
gmat-admin's picture

Question link: https://gmatclub.com/forum/a-new-law-gives-ownership-of-patents-document...

Argument summary: Even though a government may sponsor a university to do research, the government does not own the patents resulting from that research. The university owns the patents.
Logos University plans to make money by selling its patents to corporations.

Question: Which answer choice will weaken the University's plans to make money by selling its patents to corporations?

(D) Government-sponsored research conducted at Logos University for the most part duplicates research already completed by several profit-making corporations.
Aha! This tells us that the government-sponsored research isn't unique, groundbreaking research that corporations will be interested in. Instead, the government-sponsored research is simply duplicating research the corporations have already performed. So, any patents that may arise from government-sponsored research won't be any interest to corporations, since they've already conducted the same research.
This definitely weekends the University's plans to sell its patents to corporations.

(E) Logos University is unlikely to attract corporate sponsorship of its scientific research.
This has no effect on the conclusion.
Logos University is already pretty happy with its arrangement with the government. The government pays for the research, and any resulting patents belong to the university. In other words, Logos University has no interest in attracting corporate sponsorship.
As such, answer choice E has no effect on the conclusion.

Hi Brent. Again in Question - https://gmatclub.com/forum/although-the-discount-stores-in-goreville-s-central-shopping-district-50052.html,
I chose the wrong option 'A' instead of 'B'. Can you please guide me what I am doing wrong in each question? With every wrong answer I am losing patience to solve more questions and thus losing confidence. I believe that conclusion here is - 'Though Discount stores are expected to close due to SpendLess discount department store that just opened, those locations will not stay vacant for long'.

Hi Brent. Again a wrong answer chosen.
In question - https://gmatclub.com/forum/guidebook-writer-i-have-visited-hotels-throughout-the-country-and-hav-80358.html,
option 'D' looked to be strengthening the conclusion which is 'carpenters working on hotels before 1930 typically worked with more skill, care, and effort than carpenters who have worked on hotels built subsequently' so I chose 'B' as an answer but again it proved wrong. Please help me out.
gmat-admin's picture

"Option looked to be strengthening the conclusion which is 'carpenters working on hotels before 1930 typically worked with more skill, care, and effort than carpenters who have worked on hotels built subsequently"

Notice how strong this conclusion is. It says that all PRE-1930's carpenters were better than all POST-1930's carpenters

(D) The better the quality of original carpentry in a building, the less likely that building is to fall into disuse and be demolished.
This answer choice provides an alternate explanation for the connection between still-standing pre-1930's hotels and carpenters of that era.
It suggests that, if there were bad PRE-1930's carpenters, the buildings they constructed would have fallen down already.

(B) Hotels built since 1930 can generally accommodate more guests than those built before 1930.
I can see how students would choose this answer choice.
It suggests that the PRE-1930's hotels didn't have as many guests as the POST-1930's hotels, which means there was less wear and tear on the PRE-1930's hotels. This provides a different reason for why those PRE-1930's are still standing (less wear and tear).
The only problem with this is that answer choice B doesn't say the POST-1930's had MORE GUESTS than the PRE-1930's had; it says the POST-1930's had the POTENTIAL for more guests than the PRE-1930's had.
Also, the PRE-1930's were accommodating guests for years BEFORE the POST-1930's hotels were even constructed. So it's still quite possible that the PRE-1930's hotels had more guests than the PRE-1930's hotels had.

By the way, here's my solution: https://gmatclub.com/forum/guidebook-writer-i-have-visited-hotels-throug...

A greater number of newspapers are sold in Town S than in Town T. Therefore, the citizens of Town S are better informed about major world events than are the citizens of Town T.

Each of the following, if true, weakens the conclusion above EXCEPT:


(A) Town S has a larger population than Town T.

(B) Most citizens of Town T work in Town S and buy their newspapers there.

(C) The average citizen of Town S spends less time reading newspapers than does the average citizen of Town T.

(D) A weekly newspaper restricted to the coverage of local events is published in Town S.

(E) The average newsstand price of newspapers sold in Town S is lower than the average price of newspapers sold in Town T.

Hi Brent in the above question, the conclusion clearly is the fact that "Therefore, the citizens of Town S are better informed about major world events than are the citizens of Town T."

Ad the answer on GmatClub is E. I was wondering how do the number of people in either town(option A) weaken the above conclusion?
gmat-admin's picture

The information about population gives us an idea of the PROPORTION of newspaper readers in each town. Here's what I mean:

Let's say Town T has a population of 100, and 100 newspapers are sold each day in Town T.
Let's also say Town S has a population of 2,000,000, and 200 newspapers are sold each day in Town S.
This meets the condition that more newspapers are sold in Town S than in Town T.
This means 100% of the people in Town T read the newspaper, whereas only 0.01% of the people in Town S read the paper.
This would seriously undermine the conclusion that the citizens of Town S are better informed.

Does that help?

Hey Brent,
When we are attempting to find an assumption for critical reasoning questions are we looking for an assumption that supports the conclusion or goes against it?
gmat-admin's picture

If we're answering a Weaken the Argument question, then refuting an unstated assumption will weaken the argument.
Conversely, stating an unstated assumption will strengthen the argument.

Consider this example: In an attempt to make friends at school, Joe plans to give an onion to each child in his class.
One of the unstated assumptions here is that the children in his class actually want to receive an onion as a gift.

So, an answer choice that states most children don't like onions will weaken the argument, and an answer choice that suggests most children DO like onions will strengthen the argument.

In that example please let me know if this correct:
the Conclusion is the Joe is trying to make friends
Premise: He gives an onion to each child
Assumption: Joe may or may not be successful in obtaining a friend by giving an onion.
When we are trying to understand the assumption would you say its a piece of unstated evidence that has to agree with the given conclusion in the original argument? But when we look at S or W arguments we would want to change that assumption to our advantage to answer the question?
gmat-admin's picture

Sorry, the example I gave is trickier (more ambiguous) than I originally thought.

How about this example: Joe wants to make friends at school. Joe has a large supply of onions. If Joe gives onions to everyone at school, he will make lots of friends.

The conclusion here is that handing out onions will help Joe make lots of friends.
One of the unstated assumptions here is that children actually want to receive an onion.

So, an answer choice that says something like "children love onions" would strengthen the conclusion, whereas an answer choice that says "most students are terribly allergic to onions" would weaken the conclusion.

ASIDE: Don't get too caught up trying to identify assumptions. For any argument, there are tons of unstated assumptions.

The main idea here is that, answer choices that support an assumption will strengthen the argument, and answer choices that go against an assumption will weaken the argument. Also note that, arguments can be weakened and strengthened by other means as well.

Hi Brent.
Can you please help me understand this?
https://gmatclub.com/forum/in-a-certain-rural-area-people-normally-dispose-of-household-garbage-306069.html

are the GMAT mock exams questions static? If I were to do them again the questions would be the same right? in this case it is not computer adaptive?
gmat-admin's picture

The official GMATPrep practice tests pull their questions from a somewhat large collection of questions.
If you retake one of those practice tests, you will see 4 to 6 repeated questions in each section.

So should I retake them?

I already have my exam 3 and 4 purchased, I guess I will just do these new ones first
gmat-admin's picture

If you haven't already taken tests 3 and 4, I suggest you start there.
If you need additional tests, you can always retake one of them.

Hi Brent! I came across this question on GMATClub which seems to have no clear answer. I would love to have your take on this to clear my doubts. I personally see Option A as the answer since B is introducing a new element to the argument. Please help! Thank you so much in advance! :)

Link: https://gmatclub.com/forum/financial-analyst-healthcare-professionals-such-as-doctors-nurses-242623.html
gmat-admin's picture

Sorry, but I prefer not to answer unofficial Verbal questions since they often don’t represent what you’ll encounter on test day (please see comment policy)

Pages

Office Hours

On December 20, 2023, Brent will stop offering office hours. 

Change Playback Speed

You have the option of watching the videos at various speeds (25% faster, 50% faster, etc). To change the playback speed, click the settings icon on the right side of the video status bar.

Have a question about this video?

Post your question in the Comment section below, and a GMAT expert will answer it as fast as humanly possible.

Free “Question of the Day” emails!